Find f(5) if f (x) = x2 – 3x?

Answers

Answer 1
Answer: 10

Explanation:

Everywhere you see an x, replace it with 5. Then simplify using PEMDAS

f(x) = x^2 - 3x

f(5) = 5^2 - 3(5) ... plug in x = 5

f(5) = 25 - 15

f(5) = 10

Answer 2
Explanation:
f
(
5
)
=
3
(
5
)
+
2
f
(
5
)
=
17
Hopefully this helps.

Related Questions

The product of 3 and a number, divided by 9

Answers

Answer:

[tex] \frac{3(n)}{9} \: or \: 3n \div 9 [/tex]

Step-by-step explanation:

[tex] \frac{3(n)}{9} = \frac{3n}{9} [/tex]

The number is 'n'.

Hope this helps ;) ❤❤❤

There are 30 names in a hat. If two names are picked without replacement, which expression shows the probability that Jack and Jill will be picked?​

Answers

Answer: Choice C)

(1/30)*(1/29)

Explanation:

Jack has a 1/30 chance of being picked since he is 1 person out of 30 total. After his name is picked, and not put back, there are 30-1 = 29 names left. The chances Jill is picked is 1/29. The two fractions are multiplied to get the overall probability both are picked.

Twelve decreased by twice a number is -8. Find the number

Answers

Solution :-

Let the number be "x" , then according to the question :

• 2x - 12 = -8

Further solving

→ 2x = -8 + 12

→ 2x = 4 .

→ x = 2

Solution :

Let number be n.

[tex]\rule{130}{1}[/tex]

☯ [tex]\underline{\boldsymbol{According\: to \:the\: Question\:now :}}[/tex]

[tex]:\implies\sf 2n - 12 = -8 \\\\\\:\implies\sf 2n = -8 + 12\\\\\\:\implies\sf 2n = 4\\\\\\:\implies\sf n = \dfrac{4}{2}\\\\\\:\implies\underline{\boxed{\sf n = 2}}\:\:\:\:\:\Bigg\lgroup\bf{Required\: number}\Bigg\rgroup[/tex]

[tex]\therefore\:\underline{\textsf{The required number is \textbf{2}}}.[/tex]

Someone hellpppppnnnnnnnnnnn

Answers

Answer: X=11

Step-by-step explanation:

                                     7x+ 13 = 90

                                                          (bring like terms together)

                                      7x=90-13

                                      7x=77

                                           therefore x= 11

HOPE THIS HELPS YOU....

       

AND MARK THE BRAINLIEST

Flying against the wind, a jet travels 2920 miles in 4 hours. Flying with the wind, the same jet travels 7140 miles in 6 hours. What is the rate of the jet in still air and what is the rate of the wind

Answers

Answer:

Velocity in still air is = 960 miles per hour  

Velocity of wind  = 230 miles per hour.

Step-by-step explanation:

The velocity when flying against the wind = 2920 / 4 = 730 miles per hour.

The velocity when flying with the wind = 7140 / 6 = 1190

Let the rate of jet in still Air = x

Let the rate of jet in wind = y

Therefore, velocity against wind = x-y and wind = x + y

x - y = 730

x + y = 1190

Add both equation, 2x = 1920

x = 960

Now find the value of “y” = 1190 – 960 = 230

Thus, velocity in still air is = 960 miles per hour  

Velocity of wind  = 230 miles per hour.

Answer:

Rate of jet in still air = 960 miles/ hr

Rate of the wind = 230 miles/ hr

Step-by-step explanation:

Let the speed of jet in still air = [tex]u[/tex] miles/hr

Let the speed of air = [tex]v[/tex] miles/hr

So, against the wind, the resultant speed = [tex](u-v)[/tex] miles/hr

And, with the wind, the resultant speed = [tex](u+v)[/tex] miles/hr

Distance traveled against the wind = 2920 miles

Time taken against the wind = 4 hrs

Formula for distance is:

[tex]\bold{Distance =Speed \times Time}[/tex]

[tex]2920 = (u-v)\times 4\\\Rightarrow u-v=\dfrac{2920}{4}\\\Rightarrow u-v=730\ miles/hr...... (1)[/tex]

Distance traveled with the wind = 7140 miles

Time taken against the wind = 6 hrs

[tex]\bold{Distance =Speed \times Time}[/tex]

[tex]7140 = (u+v)\times 6\\\Rightarrow u+v=\dfrac{7140}{6}\\\Rightarrow u+v= 1190 \ miles/hr...... (2)[/tex]

Adding (1) and (2):

[tex]2u = 1920\\\Rightarrow \bold{u = 960 miles/hr}[/tex]

Putting [tex]u[/tex] in (1):

[tex]960 -v = 730 \\\Rightarrow \bold{v=230\ miles/hr}[/tex]

Therefore, the answer is:

Rate of jet in still air = 960 miles/ hr

Rate of the wind = 230 miles/ hr

If someone could plz help. p÷5=3.

Answers

Answer:

p=2

Step-by-step explanation:

so how to find this u take the five and three and u subtract them both and whatever is left over that is your answer

Jay just bought a new 75" television for his apartment. The opening in the entertainment center (where the tv will be placed) is only 70 inches by 35 inches. If the newer television has an aspect ratio of 16:9, will the new tv fit in the old entertainment center? Justify your answer by finding the width and height of the new television.

Answers

Answer:

Width of the new tv= 65.44 inches

Height of the new tv = 36.41 inches

If the newer television has an aspect ratio of 16:9, will the new tv fit in the old entertainment center?

Since the old entertainment set has a dimension of 70 inches by 35 inches

,new TV set, has dimension of 65.44 inches by 36.81 inches, the new TV may not be able to fit into the old entertainment set properly because it is 1.81 inches higher than the entertainment set.

Step-by-step explanation:

The newer TV set has an aspect ratio of 16: 9 and it is 75 inches

We are going to make use of Pythagoras theorem

= Width² + Height ² = Size of TV²

Width = 16x

Height = 9x

75² = (16x)² + (9x)²

5625 = 256x² + 81x²

5625 = 337x²

x² = 5625/337

x² = 16.691394659

x = √16.691394659

x = 4.0855103303

Approximately = 4.09

Hence, we can find the width and height of the new TV

Width = 16x

Width = 16 × 4.09

Width = 65.44 inches

Height = 9x

Height = 9 × 4.09

Height = 36.81 inches

Hence, the dimensions of the new TV set = 65.44 inches by 36.81 inches

If the newer television has an aspect ratio of 16:9, will the new tv fit in the old entertainment center?

Since the old entertainment set has a dimension of 70 inches by 35 inches

,new TV set, has dimension of 65.44 inches by 36.81 inches, the new TV may not be able to fit into the old entertainment set properly because it is 1.81 inches higher than the entertainment set.

Recently, the Dow opened at 26,428 ad closed at 26,340. what was the change in the Dow for the day?

Answers

Answer:

88.

Step-by-step explanation:

428-360 = 88

Find two numbers in-between 0.004 and 0.005

Answers

The 2 numbers can be: 0.0041 and 0.0045

What is the speed of a vehicle when it covers a distance of 60m in 3
seconds? *

Answers

Answer:

Hey there!

Thus, the vehicle can cover 20 m per second and would have a speed of 20 meters per second.

Let me know if this helps :)

Please help me with my homework i will give you alot of points

Answers

Answer:

see below

Step-by-step explanation:

7.    1/3 ( n+2) = 5

Multiply each side by 3

n+2 = 5*3

n+2 = 15

Subtract 2 from each side

n+2-2 = 15-2

n = 13

8.x = bills

He started with 300 and paid 1/4 of the bills and ended up with -50

300 - 1/4 x = -50

Rearranging the order on the left

-1/4x + 300 = -50

Answer:

(A) [tex]-\frac{1}{4}x+300=-50[/tex]

Step-by-step explanation:

I "reversed" this problem to find the equation that works for it.

The balance was originally $300. If he's now at a balance of -50$ after paying off [tex]\frac{1}{4}[/tex] of his bills, then [tex]\frac{1}{4}[/tex] of his bills are $350, since [tex]300 - (-50) = 350[/tex].

If  [tex]\frac{1}{4}[/tex]  of his bills are $350, then his full bill is [tex]350\cdot4=1400[/tex]$.

So I tested each equation, where [tex]x=1400[/tex] to see if the equation worked out.

For a:

[tex]-\frac{1}{4}x + 300 = -50\\\\-\frac{1}{4}\cdot1400 + 300 = -50\\\\-350 + 300 = -50[/tex]

-350 + 300 is indeed -50, so I knew this equation works.

Hope this helped!

What are the factors of x2 – 100?

Answers

Answer:

[tex] {x }^{2} - 100[/tex]

[tex] {x}^{2} - {10}^{2} [/tex]

[tex](x - 10)(x + 10)[/tex]

this is the answer

hope this is helpful for you.

Simplify -3(2x - 5)

Answers

-3 multiply by 2 is -6 and -3 multiply by -5 is 15 so the final answer is -6x+15

Please help

Identify the slope, m, and y-intercept, b, of the equation.
y = 2x + 1
Enter the correct values in the boxes.
m=1); b =

Answers

Answer:

m = 2 (the slope is 2/1), b = 1 (the y-intercept is (0,1))

Step-by-step explanation:

First, plot a point on the y-intercept (0,1). Then go up twice and to the right once to find the next point (since the slope is 2/1). You can continue doing this to form a line. The graph is shown below

Answer:

m=2

b=1

Step-by-step explanation:

We are given the equation of a line. The equation is:

y=2x+1

This equation is in slope-intercept form, or

y=mx+b  

where m is the slope and b is the y-intercept. Essentially, the coefficient of x is the slope and number that is added or subtracted from that is the y-intercept.

When we analyze the equation given, we see that x is being multiplied by 2. Therefore, the slope must be 2.

We can also see that 1 is added to 2x. Therefore, the y-intercept must be 1.

The slope of the line is 2 and the y-intercept is 1.

m=2 and b=1

The graph of the line is shown below.

Marcie solved the following inequality, and her work is shown below:

−2(x − 5) ≤ 6x + 18
−2x + 10 ≤ 6x + 18
−8x +10 ≤ 18
−8x ≤ 8
x ≤ −1

What mistake did Marcie make in solving the inequality?

Answers

Answer:

She forgot to flip the sign in the last step.

Step-by-step explanation:

In inequality, if you multiply or divide by a negative number, you would have to reverse the inequality sign.

Marice forgot to do this in the final step. She had to divide '-8' to get x completely on one side. All other steps are correct.

The correct answer is x ≥ -1.

Hope this helps.

Can someome please help me solve this my teacher hasn't done a good with teaching us this.
[tex](5.{10}^{2)} + (3 \times {10}^{4}) [/tex]

Answers

Answer:

  30500 = 3.05·10^4

Step-by-step explanation:

Your calculator can do this for you. You may need to set the display to scientific notation, if that's the form of the answer you want.

__

This can be computed by converting both numbers to standard form:

  (5·10^2) +(3·10^4)

  = 500 +30000 = 30500 = 3.05·10^4

__

Addition of numbers in scientific notation in general requires that they have the same power of 10. It may be convenient to convert both numbers to the highest power of 10.

  5·10^2 + 3·10^4

  = 0.05·10^4 +3·10^4 . . . . now both have multipliers of 10^4

  = (0.05 +3)·10^4

  = 3.05·10^4

Write the equation of the line that passes through the points (1,4) and (-7,1). Put
your answer in fully reduced point-slope form, unless it is a vertical or horizontal line.

Answers

There are two possible answers

y - 4 = (3/8)(x - 1)y - 1 = (3/8)(x + 7)

There are two answers because there are 2 points given. Both equations convert to the slope intercept form equation y = (3/8)x + 29/8

=================================================

Work Shown:

First we need the slope

m = (y2 - y1)/(x2 - x1)

m = (1-4)/(-7-1)

m = -3/(-8)

m = 3/8

-------------------

Using this slope along with the point (x1,y1) = (1,4) gets us the following equation

y - y1 = m(x - x1)

y - 4 = (3/8)(x - 1)

which is one way to express the equation in point-slope form

-------------------

Or we could easily use the other point as (x1,y1). The order of the points does not matter.

y - y1 = m(x - x1)

y - 1 = (3/8)(x - (-7))

y - 1 = (3/8)(x + 7)

is another valid answer.

what is the value of 8 + 2g - 7 when g = 5?​

Answers

Answer:

11

Step-by-step explanation:

2 times 5 = 10 so 8+10 -7= 11

The required simplified value of the given expression at g = 7 is 11.

What is simplification?

The process in mathematics to operate and interpret the function to make the function or expression simple or more understandable is called simplifying and the process is called simplification.

here,
Given expression,
y = 8 + 2g - 7

Now put g = 5,
y = 8 + 2(5) - 7
y = 8 + 10 - 7
y = 11

Thus, the required simplified value of the given expression at g = 7 is 11.

Learn more about simplification here:

https://brainly.com/question/12501526

#SPJ2

i have another one :( what is 3x - 8- 4

Answers

Answer:

3x-12 is the simplififed answer

Step-by-step explanation:

the equation is almost already simplified. when you combine -8 and -4, which are like terms, you get 3x-12. because there are no more lie temrs in the equation, it can not be simplififed any further.

Answer:

3x-12

Step-by-step explanation:

3x-8-4                        -8-4=-12

 3x-12

Which of the following is equivalent to the complex number i^13

Answers

Answer:i

Step-by-step explanatio

The number i¹³ is given by the equation A = i

What is an Equation?

Equations are mathematical statements with two algebraic expressions flanking the equals (=) sign on either side.

It demonstrates the equality of the relationship between the expressions printed on the left and right sides.

Coefficients, variables, operators, constants, terms, expressions, and the equal to sign are some of the components of an equation. The "=" sign and terms on both sides must always be present when writing an equation.

Given data ,

Let the equation be represented as A

Now , the value of A is

The complex number is n = i¹³

Now , the value of A is A = i¹³

On simplifying the equation , we get

i = √ ( -1 )

i² = -1

i³ = -i

i⁴ = 1

So ,

i¹³ = i¹² ( i )

The value of i¹² = ( i⁴ )³

The value of i¹² = ( 1 )³

The value of i¹² = 1

Now , the value of i¹³ = i¹² x i

On simplifying the equation , we get

The value of i¹³ = 1 x i

The value of i¹³ = i

Hence , the value of i¹³ = i

To learn more about equations click :

https://brainly.com/question/19297665

#SPJ2

Find the length of arc RQ. Use 3.14
for ti. Round to the nearest tenth.
6.48 m
789
1500 650
(R
[ ? Jin

Answers

Answer:

7.4

Step-by-step explanation:

NOT 8.1 !!

The length of arc RQ is 7.35 inch.

What is Length of Arc?

The distance between two places along a segment of a curve is known as the arc length.

Arc Length Formula (if θ is in degrees) s = 2 π r (θ/360°)

Given:

Angle = 65

radius = 6.48 inch

So, length of Arc

= 2πr( [tex]\theta[/tex]/ 360)

= 2 x 3.14 x 6.48 (65/360)

= 40.6944 x 0.1805

= 7.3453392

Hence, the length of arc is 7.35 inch.

Learn more about Arc Length here:

https://brainly.com/question/16403495

#SPJ2

Please help. 5. Give an example of an open equation. (1 point)

Answers

Answer:

x+4 = 8

(❁´◡`❁)(❁´◡`❁)(❁´◡`❁)

Step-by-step explanation:

An example would be x+4 = 8

(❁´◡`❁)(❁´◡`❁)(❁´◡`❁)

Prove the proposition p (0), where p (n) is the proposition "if n is a positive integer greater than 1, then n2 > n." what kind of proof did you use?

Answers

Answer:

Vacuous proof is used.

Step-by-step explanation:

Given:

Proposition p(n) :

"if n is a positive integer greater than 1, then n² > n"

To prove:

Prove the proposition p (0)

Solution:

Using the proposition p(n) the proposition p(0) becomes:

p(0) = "if 0 is a positive integer greater than 1, then 0² > 0"

The proposition that "0 is a positive integer greater than 1" is false

Since the premises "if 0 is a positive integer greater than 1" is false this means the overall proposition/ statement is true.

Thus this is the vacuous proof which states that:

if a premise p ("0 is a positive integer greater than 1") is false then the implication or conditional statement p->q ("if n is a positive integer greater than 1, then n² > n") is trivially true.

So in vacuous proof, the implication i.e."if n is a positive integer greater than 1, then n2 > n." is only true when the antecedent i.e. "0 is a positive integer greater than 1" cannot be satisfied.

A rectangle's length is 8 and the width is x + 3. Find the area.

Answers

Answer:

8x + 24

Step-by-step explanation:

Length = 8

Width = x + 3

Area of the rectangle = 8(x + 3)

= 8x + 24

1. Draw the graph of each of the following linear equations in two variables
(1) x+y=4 (ü) x - y=2
2 Give the equations of two lines passing through our​

Answers

The answer is b I just did it on the test hope it helps

The weight of meteorite A is 2 times the weight of meteorite B. If the sum of their weights is 69 ​tons, find the weight of each.

Answers

Answer:

Meteorite A= 46  Meteorite B = 23

Step-by-step explanation:

We can solve this problem by making an equation in which we add the weights, A + B = 69

We are told that A is equal to 2 times the weight of meteorite B, in other words, A is equal to 2B

We can make this into an equation and solve for B'

A + B = 692B + B = 69            Substitute A for 2B3B = 69                   Add like terms[tex]\frac{3}{3}[/tex] B = [tex]\frac{69}{3\\} \\[/tex]                   Divide each side by 3 to isolate the BB= 23

In order to solve for A, we simply input B=23 into 2B

A = 2BA = 2(23)A = 46

Check:

A + B = 69

46+23 = 69

69 = 69

factorise the polynomial X square + X - 10...what is the answer please solve fast...I will mark u brainliest but no wrong answer please ​

Answers

Answer:

(x + [tex]\frac{1}{2}[/tex] + [tex]\frac{\sqrt{41} }{2}[/tex] )(x + [tex]\frac{1}{2}[/tex] - [tex]\frac{\sqrt{41} }{2}[/tex] )

Step-by-step explanation:

solve the polynomial for x then express in product of factor form

Solve

x² + x - 10 = 0 ( add 10 to both sides )

x² + x = 10

Using the method of completing the square

add ( half the coefficient of the x- term )² to both sides

x² + 2([tex]\frac{1}{2}[/tex] )x + [tex]\frac{1}{4}[/tex] = 10 + [tex]\frac{1}{4}[/tex]

(x + [tex]\frac{1}{2}[/tex] )² = [tex]\frac{41}{4}[/tex] ( take the square root of both sides )

x + [tex]\frac{1}{2}[/tex] = ± [tex]\sqrt{\frac{41}{4} }[/tex] = ± [tex]\frac{\sqrt{41} }{2}[/tex] ( subtract [tex]\frac{1}{2}[/tex] from both sides )

x = - [tex]\frac{1}{2}[/tex] ± [tex]\frac{\sqrt{41} }{2}[/tex]

Thus corresponding factors are

(x - (- [tex]\frac{1}{2}[/tex] - [tex]\frac{\sqrt{41} }{2}[/tex] ) ) and (x - (- [tex]\frac{1}{2}[/tex] + [tex]\frac{\sqrt{41} }{2}[/tex] ), that is

(x + [tex]\frac{1}{2}[/tex] + [tex]\frac{\sqrt{41} }{2}[/tex] ) and ( x + [tex]\frac{1}{2}[/tex] - [tex]\frac{\sqrt{41} }{2}[/tex] )

Thus

x² + x - 10 = ( x + [tex]\frac{1}{2}[/tex] + [tex]\frac{\sqrt{41} }{2}[/tex] )(x + [tex]\frac{1}{2}[/tex] - [tex]\frac{\sqrt{41} }{2}[/tex] ) ← in factored form

PLZ HELP I will give the brain!! Which of the following statements is true about completing the​ square? 1. When completing the square, add the square of half the coefficient of x to only the x^2+bx side of the equation. 2. When completing the square, add the square of half the coefficient of x to both sides of the equation. 3. When completing the square, subtract the square of half the coefficient of x from both sides of the equation. 4. When completing the square, add half the coefficient of x to both sides of the equation.

Answers

Answer:

Option 2

Step-by-step explanation:

When completing the square, you have to find the number that will allow you to find the perfect square trinomials and when you factor that it should get the form a^2 + 2ab + b^2 and if a is equal to 1, then to find b, you have to take the half of the coefficient and square it because of the equation, and to maintain a balanced equation, you have to add it to both sides (same to both sides)

Hope this helped.

Answer:

[tex]\huge \boxed{\mathrm{Option \ 2 }}[/tex]

[tex]\rule[225]{225}{2}[/tex]

Step-by-step explanation:

When completing the square, we add the square of half the coefficient of x to both sides of the equation.

[tex]\Longrightarrow \ \displaystyle ( \frac{b}{2} ) ^2[/tex]

Where b is the coefficient of x in the equation.

[tex]\rule[225]{225}{2}[/tex]

If the temperature starts at 40 degrees and increases 10 degrees, what
will the final temperature be? *
+ +
-40 -35 -30 -25 -20 -15 -10 -5 0 5 10 15 20 25 30 35 40 45 50
-40
O -50
O 50
O 10

Answers

Answer:

50 degree Celsius

Step-by-step explanation:

40+10 = 50

(4 + 2i) + (-3 – 5i) show your work

Answers

Answer:

[tex] \boxed{ \bold{ \huge{\boxed{ \sf{ - 3i + 1}}}}}[/tex]

Step-by-step explanation:

[tex] \sf{(4 + 2i) + ( - 3 - 5i)}[/tex]

When there is a ( + ) in front of an expression in parentheses, there is no need to change the sign.

That means the expression remains the same.

Just remove the parentheses

[tex] \sf{4 + 2i - 3 - 5i}[/tex]

Collect like terms

⇒[tex] \sf{ - 3i + 4 - 3}[/tex]

Calculate the difference

⇒[tex] \sf{ - 3i + 1}[/tex]

Hope I helped!

Best regards!!

9514 1404 393

Answer:

  1 -3i

Step-by-step explanation:

For purposes of addition or subtraction, the i can be treated as though it were a variable. Terms can be collected in the usual way.

  4 +2i +(-3 -5i) = 4 +2i -3 -5i

  = (4 -3) +i(2 -5) = 1 -3i

Other Questions
160 is what % of the sum of 100 and 120 and 160 What is the volume of a liquid with a density of 1.5 g/L and a massof 78 grams? I need help starting off on each (3)paragraph. Im not very good at writing stuff like this. Amari is setting up a fenced-in area for his dog. The space is a square with an area of 64 feet squared. How long will one side of the fence be Listed below are several terms and phrases associated with asuumptions, board accounting principles, and coctrains. Pair each item from List A with the item from List B that is most appropriately associated with it. (a) A common denominator is the dollar. (b) All information that could affect decision should be reported. (c) Concerns the relativr size of an item and its effect on decisions. (d) Criteria usually satisfied for products at point of sale. (e) Record expenses in the period the related revenue is recognized. (f) The enterprise is separate from its owners and other entities. (g) The entity will continue indefinitely. (h) The life of an enterprise can be divided into artificial time periods. (i) The original transaction value upon acquisition.1. Expense recognition2. Periodicity3. Historical cost principle4. Materiality5. Revenue recognition6. Going concern assumption7. Monetary unit assumption8. Economic entity assumption9.Full-disclosure principle What country in Asia was mentioned as having the freedom of religion as a right?A. North KoreaB. RussiaC. ChinaD. South Korea How do contemporary issues and culture shape your worldview? Given that A={1,2,3,4,5,6,7,8} and B={2,4,6,8,10} List the elements of i.A N B. ii. A U B. The president of Deal Corporation wrote to Boyd, offering to sell the Deal factory for $300,000. The offer was sent by Deal on June 5 and was received by Boyd on June 9. The offer stated it would remain open until December 20. The offer: please help! Thank you Find the median for Andy's test scores: 76, 84, 93, 67, 82, 87. * Is -4,488/24 positive or negative? Explain in your own words the interaction between the electric and magnetic fields that make up a light wave. For the year ending December 31, 2018, Sparks Electric expects net cash provided by operations of , net cash used by investing activities of , and net cash provided by financing activities of . Sparks plans to spend to add a warehouse for its operations and pays in cash dividends. What is the amount of free cash flow? Solve the given differential equation by using an appropriate substitution. The DE is a Bernoulli equation. x dy/dx + y = 1/y^2 ________________ are substances that break lipids (fats) into very small globules so that they are more manageable in watery fluids perform the indicated operation. (4 - 8i)(2 5i) Jocko has a high school diploma and owns a corporation that nets over a million dollars a year. In view of this, what is the best description of Jocko based on middle-class standards? Find the prime factorization of the number 26 What's the present value of a perpetuity that pays $250 per year if the appropriate interest rate is 5%